LSAT and Law School Admissions Forum

Get expert LSAT preparation and law school admissions advice from PowerScore Test Preparation.

 prep88
  • Posts: 37
  • Joined: Jan 20, 2015
|
#18589
This helps, thanks!
 vedan22
  • Posts: 7
  • Joined: Apr 28, 2017
|
#34951
Hi - I'm not sure why Answer Choice C is wrong, as it seems to show the effect (snoring) without the cause (smoking). I didn't pick answer choice D because the premise in the stimulus didn't require smoking to snore, so it didn't weaken the final conclusion. Furthermore, I didn't choose A because it's not clear that stress causes the snoring independently from smoking. I read A to be stress :arrow: smoking while simultaneously stress :arrow: snoring, which wouldn't necessarily weaken the argument.
 Emily Haney-Caron
PowerScore Staff
  • PowerScore Staff
  • Posts: 577
  • Joined: Jan 12, 2012
|
#34983
Hi vedan,

Thanks for explaining your reasoning - it really makes a big difference in allowing us to give you an answer most targeted to your approach to the question!

Here, the stimulus doesn't say smoking is the ONLY cause of snoring, just that it is one cause, so C doesn't challenge that. It makes it clear that smoking must not be the only cause of snoring (since people can snore without smoking), but doesn't give us any reason to think smoking isn't ONE cause of snoring.

Your logic about A misses that it says stress induces (i.e., causes) both smoking and snoring. That means that it isn't that A causes smoking which causes snoring, or at least doesn't only mean that. Also, keep in mind that you're not looking for a perfect answer, you're looking for the best answer out of the choices given.

Does that help?
 vedan22
  • Posts: 7
  • Joined: Apr 28, 2017
|
#35475
Yes! Thanks!
 jessicamorehead
  • Posts: 84
  • Joined: Jul 07, 2017
|
#37398
I thought answer choice D is the "same cause, no effect" weakening rule. Why is A correct over D? Also what is the difference between C and D?
 nicholaspavic
PowerScore Staff
  • PowerScore Staff
  • Posts: 271
  • Joined: Jun 12, 2017
|
#37696
Hi Jessica,

Good question. You are right to be focusing on causality. So great job there!

Answer Option (C) and (D) are two different sides to the same coin. The problem comes with the phasing in both of "Most (x) do not (y)." That does not address the author's conclusion that smoking might cause snoring. It only suggests that some of the group does not show the symptom, however, isn't it still possible that smoking might cause snoring in some people? In other words, has either Answer Option (C) or (D) truly attacked the author's conclusion? Here Answer (A) is correct because it used the word "inducing." That's offering us alternative causation and that's why it's right.

Thanks for asking and I hope that clears it up! :-D
 jessamynlockard
  • Posts: 42
  • Joined: Jan 15, 2018
|
#44713
Hi,
When I was blind reviewing this question after getting it wrong, I thought D, which shows the cause without the effect was a good answer. My course stresses that showing the cause without the effect is a way to undermine a causal relationship, so could you give us some rules or reasoning about why this method isn't useful for this question type?

Thanks!
 Emily Haney-Caron
PowerScore Staff
  • PowerScore Staff
  • Posts: 577
  • Joined: Jan 12, 2012
|
#44724
Hi jessamynlockard,

Great question. Here, the trick is to really focus on what effect it would have if D were true. If most smokers do not snore, does that mean that smoking can't cause snoring in some subset of people? For example, maybe most smokers do not snore, but all smokers who had preexisting asthma snore; wouldn't it still be possible that smoking can cause snoring? Additionally, D is really just restating information from the first premise: snoring isn't common among smokers.

You might also find the post from Lucas Moreau, above, helpful as well. :)
 jessamynlockard
  • Posts: 42
  • Joined: Jan 15, 2018
|
#44725
Thanks so much! I had read the post by Lucas Moreau, but I was still having some trouble getting it to "click".
 cmorris32
  • Posts: 92
  • Joined: May 05, 2020
|
#75836
Hi PowerScore!

I read some of the questions and answers above about answer choice C, but I just wanted to clarify one thing...

When I completed this question, I had difficulty eliminating answer choice C because it shows that the effect can occur without the cause, which is one of the ways to attack a causal argument. However, I eventually eliminated answer choice C because the stimulus says that "smoking by itself CAN induce snoring." I eliminated C because the language of "can" made me think that the author used partial/possible/probable causal reasoning in this situation by saying that smoking alone can induce snoring, but it leaves open the possibility that other things unrelated to smoking can also induce snoring.

Therefore, answer choice A is correct because one of the ways to weaken possible and probable causal reasoning is to raise a 3rd possible cause that could have caused both elements. Is this correct?

Thank you!
Caroline :-D

Get the most out of your LSAT Prep Plus subscription.

Analyze and track your performance with our Testing and Analytics Package.